Find the measure of each indicated angle to the nearest degree. a.)30 b.)56 C.)50 d.)34

Find The Measure Of Each Indicated Angle To The Nearest Degree. A.)30 B.)56 C.)50 D.)34

Answers

Answer 1

Step 1

Name the sides of the triangle.

Hypotenuse = 6, the side facing right angle

Opposite = ? side facing given angle

Adjacent = 5

Step 2

Apply the cosine angle formula to find the indicated angle.

\(\begin{gathered} Cos\theta\text{ = }\frac{adjacent}{\text{Hypotenuse}} \\ \cos \theta\text{ = }\frac{5}{6} \\ \cos \theta\text{ = 0.8333333333333333333333} \\ \theta\text{ = }\cos ^{-1}0.8333333333333333 \\ \theta=\text{ }33.55 \\ \theta=\text{ 34} \end{gathered}\)

The measure of the indicated angle = 34


Related Questions

Pls help will mark brainliest fast.

Pls help will mark brainliest fast.

Answers

Answer:

7=857074/5=15/4

Step-by-step explanation:

Answer:

B

Step-by-step explanation: -x-10

which expressions are equivalent to 3^4/9/3^2/9? select all that apply

which expressions are equivalent to 3^4/9/3^2/9? select all that apply

Answers

Answer:

first and third expressions

Step-by-step explanation:

using the rule of exponents

\(\frac{a^{m} }{a^{n} }\) = \(a^{m-n}\)

then

\(\frac{3^{\frac{4}{9} } }{3^{\frac{2}{9} } }\)

= \(3^{\frac{4}{9}-\frac{2}{9} }\) ← first expression

= \(3^{\frac{2}{9} }\) ← third expression

create three equations for quadratics in vertex form that have roots at 3 and 5 but have different maximum and/or minimum values.

Answers

The three quadratic equation with different maximum and/or minimum values are y = (x-4)² – 1, y = -(x-4)² + 1, and y = -3(x-4)² + 3.

Quadratic equation:

In math, quadratic equation is the algebraic equation of the second degree in x.

The general form of quadratic equation is

ax² + bx + c = 0,

where a and b are the coefficients, x is the variable, and c is the constant term. The first condition for an equation to be a quadratic equation is the coefficient of x² is a non-zero term(a ≠ 0).

Given,

Here we need to find the three equations for quadratics in vertex form that have roots at 3 and 5 but have different maximum and/or minimum values.

As per the definition of quadratic equation, here we have to write the quadratic equation, that have the roots of 3 and 5,

Here the problem was going to call for two equations but then we have realized that it could be as simply as multiplying the a and c values by -1. Therefore, the equation are,

=> y = (x-4)² – 1,

=> y = -(x-4)² + 1,

=> y = -3(x-4)² + 3.

To know more about quadratic equation here.

https://brainly.com/question/17177510

#SPJ4

Which of the following expressions represents the solution to the inequality statement?-x ≤ -7 / x ≥ 7 / x ≤ 7 / x ≥ -7 / x ≤ -7

Answers

Answer:

x≥7

Step-by-step explanation:

mark me brainliest please

X>7 But under the > is _

Part of the pride of being an American comes from the fact that most people that want jobs have jobs. However, this is not always the case. Sometimes, as happened in 2008, the economy slows down, and jobs become harder to find or keep. Businesses make less money and so can keep fewer people employed. Those people who lost their jobs then have less money to spend. Because they have less money to spend, other businesses make less money, and then they, too, have to cut workers. In the United States, these cycles always end and the economy recovers, but it can take a while. People have to get creative about making money if they lose their jobs. Sometimes, they can use the skills they used at their old jobs to make money independently. For example, people don’t buy many car stereos when the economy is bad. So, car stereo stores have to layoff some of their stereo installers. If an installer has a place to work, that person can install stereos without a store. Often, this person will charge less than the store where he or she used to work. Also, the installer will get to keep all of the money he or she charged. At the store where the installer used to work, the store would have made most of the money. During hard times, it is important to be open-minded. There are always ways to make money outside of a formal job, if a person is willing to do things he or she wouldn’t normally do. A computer programmer, for example, might not be able to find work as a computer programmer. However, if the programmer is good with his or her hands, he or she may be able to find work in construction. Lawns never stop growing, and many aspects of life do not stop just because the economy has slowed down. A person who lost a job at a supermarket may be able to mow lawns for money, or do many other things. Most importantly, a person who loses a job must not give up. One’s attitude is very important in determining if he or she will be successful. A hopeful person is more likely to find a new job. Also, a hopeful person will

Answers

The sentence which supports the idea that people can use the skills they used at their old jobs to make money independently is "If an installer has a place to work, that person can install stereos without a store."

The correct answer option is option C

A summary about employees and American economy down time

The essay is about American employees where people who actually needs a job gets a job. Although, this is not always the case especially when the economy slows down.

This slow down in the economy causes businesses to make less profit, thereby, making most people to lose their jobs. The purchasing power of the unemployed (people who lost their job) will reduce causing a decrease in the amount of money made by businesses. This is like a vicious cycle.

However, people who are skilled can still earn a living by doing other available jobs or those who are creative can create new jobs for themselves. For instance, an accountant can become a sales person in a supermarket if he or she lost their job.

In conclusion, a person who lost their job must be determined, hopeful and hard-working and must never give up, most importantly, they should never be idle because an idle mind is the devil's workshop.

Complete question:

Which sentence supports the idea that people can use the skills they used at their old jobs to make money independently?

Answer choices

A"So, car stereo stores have to layoff some of their stereo installers."

B"Often, this person will charge less than the store where he or she used to work."

C. "If an installer has a place to work, that person can install stereos without a store."

D. "At the store where the installer used to work, the store would have made most of the money."

Read more on supporting sentences:

https://brainly.com/question/12809344

#SPJ1


Saturday Sunday
Minutes
32
Miles
3
4
I

Answers

it’s droopy dog B homie it’s B

I NEED HELP PLS HURRY

1. Write each expression with a single exponent:
a. (10^7)²
b. (10^9)³
c. (10^6)³
d. (10^2)³
e. (10³)²
f. (10^5)^7

Answers

a. 10^14
b. 10^27
c. 10^18
d. 10^6
e. 10^6
f. 10^35

multiply the exponents.

Answer:

We use the rule,

\((a^b)^c = a^{bc}\)

a. (10^7)²

\((10^7)^2 = 10^{(7)(2)} = 10^{14}\)

10^14

b. (10^9)³

\((10^9)^3 = 10^{(9)(3)} = 10^{27}\)

10^27

c. (10^6)³

\(10^{(6)(3)}= 10^{18}\)

10^18

d. (10^2)³

\(10^{(2)(3)} = 10^{6}\)

10^6

e. (10³)²

\(10^{(3)(2)}=10^{6}\)

10^6

f. (10^5)^7

\(10^{(5)(7)} = 10^{35}\)

10^35

Step-by-step explanation:

1. How many variables are involved in the chi-square test?
2. Which type of chi-square test is this?
A.goodness of fit B.test of independence
3. How many degrees of freedom are involved?
4. Using the chi-square critical values table is the result of this rest statistically significant?
A. Yes B.No

1. How many variables are involved in the chi-square test? 2. Which type of chi-square test is this?

Answers

The result of the respective questions are:

This chi-square test only takes into consideration one variable.The  type of chi-square test this is is a Goodness of Fitdf= 3NO

How many variables are involved in the chi-square test?

a)

This chi-square test only takes into consideration one variable.

b)

The  type of chi-square test this is, is a Goodness of Fit

To test the hypothesis, we must determine whether the actual data conform to the assumed distribution.

The "Goodness-of-Fit" test is a statistical hypothesis test that determines how well the data that was seen resembles the data that was predicted.

c)

Parameter

n = 4

Therefore

Degrees of freedom

df= n - 1

df= 4 - 1

df= 3

d)

In conclusion

Parameters

\(\alpha = 0.05\)

df = 3

Hence

Critical value = 7.814728

Test statistic = 6.6

Test statistic < Critical value, .

NO, the result of this test is not statistically significant.

Read more about Probability

https://brainly.com/question/11234923

#SPJ1

Select the correct answer.
Each statement describes a transformation of the graph of f(x) = x. Which statement correctly describes the graph of g(x) if g(x) = f(x - 11)?
A. It is the graph of f(x) where the slope is increased by 11.
It is the graph of f(x) translated 11 units to the left.
It is the graph of f(x) translated 11 units up.
It is the graph of f(x) translated 11 units to the right.
B.
C.
OD.

Answers

The correct answer is C. It is the graph of f(x) translated 11 units to the left.

The correct answer is:

C. It is the graph of f(x) translated 11 units to the left.

When we have a function of the form g(x) = f(x - a), it represents a horizontal translation of the graph of f(x) by 'a' units to the right if 'a' is positive and to the left if 'a' is negative.

In this case, g(x) = f(x - 11), which means that the graph of f(x) is being translated 11 units to the right. However, the answer options do not include this specific transformation. The closest option is option C, which states that the graph of g(x) is translated 11 units to the left.

The graph of f(x) = x is a straight line passing through the origin with a slope of 1. If we apply the transformation g(x) = f(x - 11), it means that we are shifting the graph of f(x) 11 units to the right. This results in a new function g(x) that has the same shape and slope as f(x), but is shifted to the right by 11 units.

Therefore, the correct answer is C. It is the graph of f(x) translated 11 units to the left.

for more such question on graph visit

https://brainly.com/question/19040584

#SPJ8

3
Find the value of x.
34°
o
27°

3Find the value of x.34o27

Answers

Hi there! :)

\(\large\boxed{x = 122^{o}}\)

Recall that the sum of all angles in a triangle = 180°.

Therefore:

34° + 24° + x° = 180°

Combine like terms and subtract:

58° + x° = 180°

x = 180° - 58°

x = 122°

Answer:

x = 39

Step-by-step explanation:

27+34 = 61 100-61 = 39

Which experience is most likely to shape an author's perspective on
vegetarianism?

Which experience is most likely to shape an author's perspective onvegetarianism?

Answers

Answer:

I would say working on a farm. Farms include growing and harvesting vegetables. So they might be biased to one side of vegetarianism. Hope this helps

Answer: working on a farm

Step-by-step explanation: I did the quiz

Shape A is transformed to shape B by a rotation of 90° anti-clockwise.
Using a pair of compasses, find the centre of rotation.

Answers

The intersection of these two perpendicular bisectors is the center of rotation.

How to find the center of rotation?

We need to identify a point on the shape that does not move during the transformation.

One way to do this is to identify a point on Shape A and its corresponding point on Shape B, and then find the perpendicular bisector of the line segment connecting these two points. The center of rotation will be the intersection of all such perpendicular bisectors.

Draw a line segment connecting a point on Shape A to its corresponding point on Shape B. Label the two endpoints of the line segment as A and B, respectively.

With the compasses set to any radius greater than half the length of AB, draw arcs centered at A and B that intersect at two points on either side of the line segment. Label these points as C and D, respectively.

With the same radius, draw arcs centered at C and D that intersect each other at two points. Label these points as E and F, respectively.

Draw the perpendicular bisectors of line segments AC and BD. These are the lines that pass through the midpoints of AC and BD and are perpendicular to them.

The intersection of these two perpendicular bisectors is the center of rotation.

Note that the center of rotation may not be inside the shapes themselves, but outside them. Also, you can repeat this process with different pairs of corresponding points to check if the center of rotation is consistent.

Learn more about center of rotation, visit

brainly.com/question/29439290

#SPJ9

5. El bloque de la figura se encuentra en reposo en el momento en que empiezan a actuar las fuerzas.
F₁ = 25 N
F, = 22.6 N
m = 2 kg
a. Encontrar el valor de la fuerza resultante.
b. Determina su aceleración.
C. Calcula la velocidad del bloque a los 5 segundos de movimiento.

Answers

The resultant force acting on the block is 47.6N.

The acceleration of the block will be 23.8 m/s².

The speed of the block after 5 seconds of movement is 119 m/s.

What is the resultant force?

In this question, we need to make use of Newton's second law, hence:

resultant force = mass × acceleration

So, for question a. The value of the resultant force  will be obtained by adding the two forces together:

resultant force = F₁ + F₂

                       = 25N + 22.6N

                        = 47.6N

b.  Its acceleration will be:

acceleration = resultant force / mass

                  = 47.6N / 2kg

                  = 23.8 m/s²

c. The speed of the block after 5 seconds of movement will be:

Velocity  (v) = u + at

velocity = initial velocity + acceleration x time

Based on the block was initially at rest, the initial velocity is 0.

Hence:

V = acceleration x time

    = 23.8 m/s² x 5s

     = 119 m/s

Hence the speed of the block after 5 seconds of movement will be: 119 m/s.

Learn more about resultant force  from

https://brainly.com/question/25239010

#SPJ1

See full text below

The block in the figure is at rest when the forces begin to act.

F₁ = 25N

F, = 22.6 N

m = 2kg

to. Find the value of the resultant force.

b. Determine its acceleration.

C. Calculate the speed of the block after 5 seconds of movement.

Find the length of m

Anybody can help please?

Find the length of mAnybody can help please?

Answers

Answer:

m = 10

Step-by-step explanation:

We are going to use sine ratio as sine ratio is opposite to hypotenuse.

We know the value of opposite which is 5√3

The value of hypotenuse is m which is unknown.

Therefore:-

\( \displaystyle \large{ \sin(60 \degree) = \frac{5 \sqrt{3} }{m} }\)

We know that sin60° is √3/2

\( \displaystyle \large{ \frac{ \sqrt{3} }{2} = \frac{5 \sqrt{3} }{m} }\)

Multiply both sides by LCM which is 2m.

\( \displaystyle \large{ \frac{ \sqrt{3} }{2} (2m)= \frac{5 \sqrt{3} }{m} (2m)} \\ \displaystyle \large{ \sqrt{3} m=10 \sqrt{3} }\)

Divid both sides by √3 to isolate m.

\( \displaystyle \large{ \frac{ \sqrt{3}m }{ \sqrt{3} } = \frac{10 \sqrt{3} }{ \sqrt{3} } } \\ \displaystyle \large{ m = 10}\)

And we're done! The value of m is 10.

Alternative Solutions

If we do not want to use sin60°, we can use cos30°.

Focus the 30°, since for 30°, 5√3 is adjacent and m is hypotenuse.

cosine ratio is adjacent to hypotenuse.

Therefore:-

\( \displaystyle \large{ \cos(30 \degree) = \frac{5 \sqrt{3} }{m} }\)

We know that cos30° is √3/2

\( \displaystyle \large{ \frac{ \sqrt{3} }{2} = \frac{5 \sqrt{3} }{m} }\)

Notice something? Both equations when we use sin60° and cos30° are same. This is called a co-function.

Since sin60° = cos30°, both methods work.

If we do not want to use sin60°, you can use cos30°.

the function has zeros at -1 and -11, and a minimum at -5

Answers

The minimum point on the graph is \((-6,25)\), which confirms that the function has a minimum at\(-6\).

What is graph?

In mathematics, a graph is a diagram that shows the relationship between different sets of data. It is made up of points, which are also called vertices or nodes, that are connected by lines or curves called edges or arcs.

In mathematics, a function is a relation between two sets of data, such that each input in the first set corresponds to exactly one output in the second set. In other words, a function maps each input value to exactly one output value.

According to given information:

Let's start by writing the quadratic function in factored form, given that it has zeros at \(-1\) and \(-11\):

\(f(x) = a(x- (-1))(x- (-11))\)

Simplifying, we get:

\(f(x) = a(x+1)(x+11)\)

To find the value of a, we need to use the fact that the function has a minimum at \(-5\). Since the vertex of the parabola is at the minimum point, we know that the x-coordinate of the vertex is \(-5\). Therefore, we can use this information to find the value of a as follows:

\(-5 = (-1+(-11))/2\\-5 = -6\)

This tells us that the axis of symmetry of the parabola is \(x =-5\). Since we know that the function has zeros at \(-1\) and \(-11\), we can deduce that the vertex must lie halfway between these two zeros, at\(x = -6\). Therefore, the value of a is:

\(f(-6) = a((-6)+1)(-6) +11) = a(5) (-1) = -5a\)

We also know that the function has a minimum at this point, so we can use this to find the value of a:

\(f(-6) = -5\\-5= -5a\\\\a = 1\)

Therefore, the quadratic function that satisfies these conditions is:

\(f(x) = (x+1) (x+11)\)

We can check that this function has zeros at \(-1\) and \(-11\), and that it has a minimum at\(x =-6\) by finding its vertex:

x-coordinate of vertex = \((-1 +(-11))/2 =-6\)

y-coordinate of vertex =\(f(-6) = (-6+1)(-6+11) = 25\)

Therefore, the minimum point on the graph is \((-6,25)\), which confirms that the function has a minimum at -6.

Which of the following function has zeros at \(-1\) and\(-11\) and a minimum of \(-5\) ?

To know more about graph visit:

https://brainly.com/question/19040584

#SPJ1

Consider the hospital emergency room data displayed below. Let A denote the event that a visit is to hospital 4, and let B denote the event that a visit results in LWBS (at any hospital). Determine the following probabilities. Round your answer to three decimal places (e.g. 98.765). Hospital1 2 3 4 Total LWBS 199 275 243 249 966Admitted 1274 1551 662 989 4476Not adted 3830 5164 4724 3105 16823 Total 5303 6990 5629 4343 22265(a) P(A | B) =(b) P(A' I B) =(c) P(A | B') =(d) P(B | A) =

Answers

(a) P(A | B) = P(A and B) / P(B) = 249 / 966 = 0.257

(b) P(A' I B) = P(B) - P(A and B) = 966 - 249 = 717

(c) P(A | B') = P(A and B') / P(B') = (4343 - 249) / (22265 - 966) = 3993 / 21399 = 0.187

(d) P(B | A) = P(A and B) / P(A) = 249 / 989 = 0.252

Emergency Room Probabilities Calculation

(a) P(A | B) = P(A and B) / P(B) = 249 / 966 represents the probability of a visit resulting in LWBS given that the visit was to hospital 4. The numerator is the number of visits to hospital 4 that resulted in LWBS, and the denominator is the total number of visits that resulted in LWBS at any hospital.

(b) P(A' I B) = P(B) - P(A and B) represents the probability of a visit resulting in LWBS given that the visit was NOT to hospital 4. The numerator is the total number of visits that resulted in LWBS at any hospital minus the number of visits to hospital 4 that resulted in LWBS, and the denominator is not needed because it is a constant (1).

(c) P(A | B') = P(A and B') / P(B') represents the probability of a visit being to hospital 4 given that the visit did NOT result in LWBS. The numerator is the number of visits to hospital 4 that did NOT result in LWBS, and the denominator is the total number of visits that did NOT result in LWBS at any hospital.

(d) P(B | A) = P(A and B) / P(A) represents the probability of a visit resulting in LWBS given that the visit was to hospital 4. The numerator is the number of visits to hospital 4 that resulted in LWBS, and the denominator is the total number of visits to hospital 4.

Learn more about Emergency Room Probabilities Calculation here:

https://brainly.com/question/28758605

#SPJ4

Type the correct answer in the box. Use numerals instead of words. If necessary, use / for the fraction bar.
A/60
B/45
C/105
The measurement of angle A is
The measurement of angle B is
The measurement of angle Cis

Answers

The second pair of points representing the solution set of the system of equations is (-6, 29).

To find the second pair of points representing the solution set of the system of equations, we need to substitute the x-coordinate of the second point into one of the equations and solve for y.

Given the system of equations:

y = x^2 - 2x - 19

y + 4x = 5

Substituting the x-coordinate of the second point (-6) into equation 2:

y + 4(-6) = 5

y - 24 = 5

y = 5 + 24

y = 29

Therefore, the second pair of points representing the solution set of the system of equations is (-6, 29).

for such more question on system of equations

https://brainly.com/question/4262258

#SPJ8

Question

Type the correct answer in the box. Use numerals instead of words. If necessary, use / for the fraction bar.

y = x2 − 2x − 19

y + 4x = 5

The pair of points representing the solution set of this system of equations is (-6, 29) and

_________.

tính z=(〖(1+i)〗^10-1)/(〖(1+i)〗^10+1)

Answers

It looks like the given complex number is

z = ((1 + i )¹⁰ - 1) / ((1 + i )¹⁰ + 1)

Let w = 1 + i, so we rewrite

z = (w ¹⁰ - 1) / (w ¹⁰ + 1)

Since w * = 1 - i, we get w w * = (1 + i ) (1 - i ) = 1 - (-1) = 2. Multiply z by ((w *)¹⁰ + 1) / ((w *)¹⁰ + 1). This gives

z = (2¹⁰ - 1 + w ¹⁰ - (w *)¹⁰) / (2¹⁰ + 1 + w ¹⁰ + (w *)¹⁰)

Now, writing w in polar form yields

w = √2 exp(i π/4)

==>   w ¹⁰ = (√2)¹⁰ exp(i 10π/4) = 2⁵ exp(i 5π/2) = 2⁵ i

Similarly,

w * = √2 exp(-i π/4)

==>   (w *)¹⁰ = -2⁵ i

So we have

w ¹⁰ + (w *)¹⁰ = 0

w ¹⁰ - (w *)¹⁰ = 2 × 2⁵ i = 2⁶ i

Then we end up with

z = (2¹⁰ - 1 + 2⁶ i ) / (2¹⁰ + 1) = (1023 + 64i ) / 1025

Find the equation of a line with the given informatio f(-6) = 9 and f (20) = 5.

Answers

The equation of the line is with the coordinates f(-6) = 9 and f (20) = 5. is y = (-2/13)x + 117/13.

What is slope-intercept form?

Y = mx + b, where m is the line's slope and b is the y-intercept, is the slope-intercept form of a linear equation. This form is helpful since it provides information about the line's slope and y-intercept in a clear and understandable manner. We can quickly determine the slope of the line (m) and its point of intersection with the y-axis by glancing at the equation (b). By beginning at the y-intercept and using the slope to determine other locations along the line, we can also graph the line using this method.

The slope of the line is given as:

m = (y2 - y1) / (x2 - x1)

Substitute the given coordinates:

m = (5 - 9) / (20 - (-6))

m = -4 / 26

m = -2 / 13

Using the point slope form we have:

y - y1 = m(x - x1)

Using the point (-6, 9):

y - 9 = (-2/13)(x - (-6))

y - 9 = (-2/13)x - 12/13

y = (-2/13)x + 117/13

So the equation of the line is y = (-2/13)x + 117/13.

Learn more about slope intercept form here:

https://brainly.com/question/29146348

#SPJ1

use the diagram below to find all missing angles

use the diagram below to find all missing angles

Answers

Answer:

Step-by-step explanation:

Soooo, angle 1 is 157 degrees

That means:

Angle 2 is 23 degrees                                   Supplementary Angles

Angle 3 is 157 degrees                                  Vertical Angle to Angle 1

Angle 4 is 23 degrees                                   Vertical angle to Angle 2

Answers:

angle 2 = 23 degreesangle 3 = 157 degreesangle 4 = 23 degrees

=======================================================

Explanation:

Angles 1 and 2 are supplementary as they form a straight line. So the angles add to 180

(angle1)+(angle2) = 180

angle2 = 180-(angle1)

angle2 = 180-157

angle2 = 23 degrees

Angle 4 is congruent to this because angles 2 and 4 are vertical angles. Vertical angles are always congruent. Note how angles 1 and 4 are supplementary.  

Since angles 1 and 3 are the other pair of vertical angles, this must mean angle 3 is 157 degrees.

need help really please​

need help really please

Answers

C and D because of the hundredths and thousandths place

Answer:

B, D

Step-by-step explanation:

The following table shows the number and type of properties available in three cities.

City
Key West
Orlando
Miami

Write a matrix that represents the number of each type of property available in Key West.

The following table shows the number and type of properties available in three cities.CityKey WestOrlandoMiamiWrite

Answers

[  3   9   13 ] is the Matrix showing all types of property available in key west .

What is matrix ?

A matrix is a rectangular array or table of numbers, symbols, or expressions arranged in rows and columns to represent a mathematical object or an attribute of such an entity in mathematics.

Calculation

given in the table that how many types of property are there in the table

its a simple matrix which u can directly see

its a  3 x 1 matrix ...

[3  9  13] this matrix shows the required value

hope this helps...

learn more about matrix here :

brainly.com/question/28180105

#SPJ13

Answer:  [3 9 13]

Step-by-step explanation:

A fair coin is to be tossed 10 times. Let i/j, in lowest terms, be the probability that heads never occur on consecutive tosses. Find i+j.

Answers

The probability that there are 89/1024 ways in which heads never happen on consecutive tosses.

Explain the definition of probability?Mathematics' study of random events is known as probability, and there are four primary types of probability: axiomatic, classical, empirical, and subjective. Since probability is the same as possibility, you could say that it is the likelihood that a specific event will occur.

For the stared question;

Think of the flips as either a string of [Head first Tail] or [Tail]. Let x stand for the quantity of [Head followed by Tail] and y for the quantity of [Tail]. If 2x + y = 10, then.

Then, for every value of x, I completed casework:

The number of configurations of AAAAAAAAAA is bijective to x=0 and is 1.

The number of configurations of AAAAAAAAB is then bijective to x=1, which really is 9 and so forth.

These variables add up to 89, and there are 1024 different ways to flip them.

P(i + j) = 89/1024

Thus, there are 89/1024 ways in which heads never happen on consecutive tosses.

To know more about the probability, here

https://brainly.com/question/13604758

#SPJ4

Question 6
Which is the graph of y=-x-3?
Graph B
Graph C
Graph A

Answers

Answer:

Step-by-step explanation:

Question 6Which is the graph of y=-x-3?Graph BGraph CGraph A

solve for u. -32=-4u simplify your answer as much as possible

Answers

Answer: u = 8

Step-by-step explanation:

We need to get the variable by itself first. In this case the variable is u.

u is really just mutliplied by -4, (-4 x u). In this situation, the opposite of  multiplication is division. So, we need to divide -4u by -4 on each side.

-32 = -4u

-32/-4 = -4u/-4

8 = u

u = 8

u=8
you divide by -4 on both sides and -32/-4 = 8

Irina plotted the number of participants in a summer camp program on a coordinate plane to create the following scatterplot. Which graph shows the line of best fit for this scatterplot?

Answers

Answer: A or the first graph on edge 2021

Step-by-step explanation: Just took the test good luck

Answer:

The first graph (A)

Step-by-step explanation:

24 boys, 45 girls and 281 adults are the members of a badminton club.
50 more children join the club.
The number of girls is now 18% of the total number of members.
How many of the 50 children were boys?

Answers

24+45+281+50=400

(45+n/400)×100=18

45+n=18×4=72

n=72-45=27

50-27=23.

hope it's correct

23 of the 50 children added were boys. The answer is 23 boys

Given that 24 boys, 45 girls and 281 adults are the members of a badminton club.

Total number of people as members of badminton = 24 + 45 + 281 = 350

Percentage of girls = girls/total number x 100

% of girls = 45/350 x 100

% of girls = 12.9%

If 50 more children join the club, and the number of girls is now 18% of the total number of members. Then, the new total number of members will be

New total members = 350 + 50 = 400

Let g = new added girls from the children

Percentage of girls = girls/total number x 100

18 = (g + 45) / 400 x 100

18/100 =  (g + 45) / 400

cross multiply

0.18 x 400 = g + 45

72 = g + 45

make g the subject of formula

g = 72 - 45

g = 27 girls

To get the number of boys added, take 27 away from the 50 children. That is,

The number of boys added = 50 - 27 = 23 boys

Therefore, 23 of the 50 children added were boys

Learn more about arithmetic here: https://brainly.com/question/6561461

Un hombre posee 50 acciones con un valor de 30 cada una , la corporación declaró un dividendo del 6% pagadero en acciones ¿cuantas acciones poseía entonces?

Answers

el hombre tenía 50 acciones.

La pregunta es, ¿cuántas acciones poseía el hombre si declararon un dividendo del 6% pagadero en acciones y tenía 50 acciones con un valor de $30 cada una?Para calcular cuántas acciones tenía el hombre,

se puede utilizar la siguiente fórmula:Dividendos = Número de acciones * Precio por acción * Tasa de dividendosDe esta fórmula,

podemos despejar el número de acciones. Así, tenemos:Número de acciones = Dividendos / (Precio por acción * Tasa de dividendos)De los datos del problema, se sabe que el hombre tenía 50 acciones con un valor de $30 cada una.

Además, la corporación declaró un dividendo del 6% pagadero en acciones. Por lo tanto, la tasa de dividendos es del 6%.Para resolver el problema, primero debemos calcular el valor del dividendo que se pagará en acciones.

Para ello, se debe multiplicar el valor de las acciones del hombre por la tasa de dividendos.

Así, tenemos:Valor del dividendo = 50 acciones * $30 * 0.06 = $90El valor del dividendo es de $90. Ahora, podemos sustituir este valor en la fórmula para calcular el número de acciones que tenía el hombre.

Así, tenemos:Número de acciones = $90 / ($30 * 0.06) = 50 accionesPor lo tanto,

To learn more about : tenía

https://brainly.com/question/29256780

#SPJ8


y = 1/3x -1

x-intercept (3,0)

How did they get this answer? Somebody please help

Answers

The x-intercept of a line is the point where the line crosses the x-axis, which means that the y-coordinate of the point is zero. To find the x-intercept of the line y = (1/3)x - 1, we need to substitute y with zero and solve for x.

Here's how to do it:

Substitute y with zero:
0 = (1/3)x - 1

Add 1 to both sides:
1 = (1/3)x

Multiply both sides by 3:
3 = x

So the x-intercept of the line y = (1/3)x - 1 is (3,0). This means that the line crosses the x-axis at the point (3,0).

In conclusion, the x-intercept of the line y = (1/3)x - 1 is (3,0), which means that the line crosses the x-axis at the point (3,0).

Answer:

Step-by-step explanation:

x-intercept is where the line cuts the x-axis. That is, when y=0.

Substitute y=0 and we get:

       \(0=\frac{1}{3} x-1\)

       \(1=\frac{1}{3} x\)

       \(x=3\)

So x-intercept is the point (3,0).

THIS IS URGENT PLEASE HELP!!!!
the smith family has a 10% off coupon for dinner at their favorite restaurant. Of the bill $42.50 and they wish to leave a 15% tip, what will they pay in total?

Answers

Answer:

The answer is 44.625 .

Step-by-step explanation:

Basically, if we have a pretend number line in your head, subtract 10%. Now, you will have -10%. Add 15%, now: you will have 5%. So, do $42.50 x 5%.

How to calculate percentage of a number. Use the percentage formula: P% * X = Y

Convert the problem to an equation using the percentage formula: P% * X = Y.

P is 10%, X is 150, so the equation is 10% * 150 = Y.

Convert 10% to a decimal by removing the percent sign and dividing by 100: 10/100 = 0.10.

The answer is 44.625  payed in total to their favorite restaurant.

Other Questions
can you help me with this assignment? a malignant tumor that originates within glandular tissue amos' antiques, inc., is characterized by separation of management from ownership and by clearly defined lines of authority and responsibility. these characteristics are consistent with the principles of MINI ESSAYExplain the following statement: "No clapping is possible without two hands to do it, and no quarrel without two persons to make it." What do you think Gandhi's purpose is in using these images? What is the Vertex of 12 and 1 The time difference, t, in minutes, taken for an object to travel from point A to B and the return journey is given by the equation (t+6) (t-8)=72. By solving the equation (t+6) (t-8)=72, find the time difference t in minutes. which of the following describes how Hoover and Roosevelt were most similar in their immigration policies? A) They both supported safe work camps for migrants traveling in search of work. B) They both lobbied Congress to favor English-speaking immigrants. C)They both lobbied Congress to set stricter limits on all immigration. D) They both supported quotas on the number of immigrants who entered the US. If f(x)=3x+2 and g(x)=x2x, find the value. f(2)= What is the name of the reaction that removes the hydrocarbon chain from the triglyceride. While shopping at a music store, Rick bought 7 CDs, all at the same price. The tax was $6.50 and the total was $66.00. What was the price of each CD? can someone pls help with this question Which of the following best explains the effects of Mongol trade policies?Mongols controlled overland trade, leading Europeans to search for alternate trade routes.Mongols avoided contact with most major religious groups in Asia to prevent assimilation.O Mongols successfully spread their culture and religious beliefs along overland trade routes.O Mongols heavily taxed seafaring traders, thus encouraging overland trade to flourish. What is the most common grip used in racket sports? Both the murderer and Lady Macduff herself call Macduff a traitor. In what scene does each mean it? Do you think Macduff is a traitor, in either sense? What is 903 divided by 72?33 POINTS FOR THIS QUESTION!! which of these was a major convern for farmers out in the west during the 1890sA. The lack of protection from attacks by native americansB. the over production of wheat from the great plainsC. The cost of shipping crops by railroad to the markets in the east Read and choose the correct word for the blank that matches the image. A room with green curtains Mi dormitorio tiene ________ verdes. los patios los tapetes las puertas las cortinas Which relationships describe angles 1 and 2?Select each correct answeradjacent anglesvertical anglescomplementary anglessupplementary angles What is sustainability? How can we make sustainable development a reality? How sustainability can be measured? S() 5(0) Problem #6: Let F(x)=f(+5()). Suppose that f(4) = 6, f'(4) = 2, and S'(12) = 3. Find F'(2). Problem #6: Just Save Submit Problem #6 for Grading Attempt 1 Problem #6 Your Answer: Your Mark: At